LSAT and Law School Admissions Forum

Get expert LSAT preparation and law school admissions advice from PowerScore Test Preparation.

 Administrator
PowerScore Staff
  • PowerScore Staff
  • Posts: 8916
  • Joined: Feb 02, 2011
|
#73428
Complete Question Explanation

Principle, Weaken. The correct answer choice is (A).

The letter writer claims that the newspaper has shown "unjustified bias" in the skeptical way it reported Mr. Hanlon's extraordinary claim that he saw an alien spaceship. The evidence is that Mr. Hanlon is a trusted member of the community and that, if his claim was of a more ordinary sort, the report would not have been so skeptical. We are asked to find a principle that conflicts with this argument, weakening it. This means the correct answer will be a rule that suggests that the newspaper's skeptical tone in their reporting may NOT have been unjustified. A good prephrase here might be "if a claim is extraordinary, skepticism in the reporting of the claim is justified."

Answer choice (A): This is the correct answer choice. This is a pretty close match to our prephrase, although some might not at first see that "uncritical" means "without skepticism." Note that in the stimulus the only evidence is that Mr. Hanlon said he saw a spaceship, and one eyewitness claiming to have seen something extraordinary is surely not "evidence of an extraordinarily high standard." This answer is, in essence, saying that the reporting in this case should have been critical or skeptical, and that undermines the claim that the skepticism was evidence of an unjustified bias.

Answer choice (B): The stimulus gives no reason to believe that any intermediary was involved in passing the information from Mr. Hanlon to the newspaper. Thus, the rule in this answer does not appear to have been violated, because the rule simply doesn't apply.

Answer choice (C): A bit of a shell game here, we have no way of knowing whether Mr. Hanlon has ever been a trusted source for the newspaper, but only that he is a trusted member of the community. But even if he had been a trusted source before, this answer would actually strengthen the claim that the newspaper was unjustified in the way they reported the story.

Answer choice (D): The rule in answer D has nothing to do with what the newspaper should or should not have done in their reporting, but is only about what Mr. Hanlon should have done. As such, it cannot weaken the argument that is about what the newspaper supposedly did wrong.

Answer choice (E): The rule in answer E is about whether the paper should have published the story at all, and tells us nothing about HOW they should have reported it. Whether the story was confirmed by an independent source or not, that would tell us nothing about how skeptical or uncritical they should have appeared in their reporting.
 jlam061695
  • Posts: 62
  • Joined: Sep 17, 2016
|
#30876
I really do not see why A is the correct answer? The principle that I extracted from the stimulus was: If a trusted member of the community reports on an event (regardless of whether it is extraordinary or natural), then a newspaper must not be skeptical of the trusted member's testimony; otherwise, the newspaper exhibits a bias. How does this conflict in any way with the principle that an extraordinary claim ought to be expressed in a skeptical way (unless there is evidence that's held to a high standard)? Is it because Mr. Hanlon's evidence, the veracity of which is based on the fact that he is a trusted member, is not held to a high standard?
User avatar
 Jonathan Evans
PowerScore Staff
  • PowerScore Staff
  • Posts: 726
  • Joined: Jun 09, 2016
|
#30899
Hi, JLam,

Let's just follow the straight conditional in Answer Choice (A).

We have established that the claim that Hanlon saw an alien spaceship is an extraordinary claim.

From the conditional in (A), this is sufficient to know that it should be presented critically.

This claim can only be presented uncritically if "backed by evidence of an extraordinarily high standard." As you noted in your post, irrespective of whether Hanlon is well respected, his testimony is not "evidence of an extraordinarily high standard." Therefore, an uncritical presentation of Hanlon's claim would necessarily violate the conditional statement in Answer Choice (A).
 mhsk
  • Posts: 7
  • Joined: Oct 28, 2017
|
#41739
Hi,

I would like to clarify how the logic needs to be diagrammed for Answer Choice A). The way Answer Choice A) is structured really confused me because there are more than one logic associated with it. Here is how I diagrammed it:

First part ("If a claim is extraordinary, it should not be presented uncritically"):
Ext :arrow: Presented Uncritically

Second part ("it should not be presented uncritically unless it is backed by evidence of an extraordinarily high standard"):
Presented Uncritically :arrow: Backed by E of EHS

So, Answer Choice A) would be diagrammed as:
Ext :arrow: Presented Uncritically :arrow: Backed by E of EHS

Answer is A) because the claim is not backed by E of EHS, and, therefore, should be presented critically (skeptical), while Hanlon's claim about meteor would not have been taken skeptically:
~Backed by E of EHS :arrow: ~Presented Uncritically (aka. Skeptical)

Could you let me know if my diagram is correct? Thanks!

-Rachel
 James Finch
PowerScore Staff
  • PowerScore Staff
  • Posts: 943
  • Joined: Sep 06, 2017
|
#41790
Hi Rachel,

Your diagramming is partially correct. The key to this question is abstracting the proper principle from the stimulus, which is that if a source has been trusted in the past, that source's claims should be presented uncritically regardless of whether they are extraordinary or not. This then conflicts with the first part of (A), which says that extraordinary claims should not be presented uncritically (or Ext. :arrow: Uncritically). The evidence of an extraordinarily high standard is a red herring, as there is nothing in the stimulus about it (a trusted source would be pretty ordinary evidence).

Hope this clears things up!
 alexmcc
  • Posts: 22
  • Joined: Aug 02, 2018
|
#49832
I skipped over A because I failed to make a mental note that having a skeptical tone in an article is the same thing as offering a critical view in an article. This makes more sense now. Thanks!
 Rachael Wilkenfeld
PowerScore Staff
  • PowerScore Staff
  • Posts: 1358
  • Joined: Dec 15, 2011
|
#53453
Hi Alex,

So glad to hear it helped! It can be hard to get some of the terminology down for this test, particularly when they use different terms to cover the same idea. The key here is that skepticism is a type of criticism, so it falls into the umbrella term used in answer choice (A). It can be very easy to overlook that answer though since it uses different phrasing than in the stimulus.

Keep up the good work.

Rachael
 michael1345
  • Posts: 3
  • Joined: Aug 31, 2018
|
#56797
hi, I understand the stimulus and why A is the correct answer but it still bugs me. A says unless its backed up by high standard evidence, it must be presented critically. Usually LR questions are much more specific and 100%, but I just dont see anywhere that proves that he did not have that high standard evidence. It just says that he claimed to have saw the spaceship. He could have backed up his claims with science, or had some alien detecting technology for all we know. Therefore the principle wouldnt necessarily conflict with the argument. I know it was very specific of me to mention this but i thought that was what the LSAT was...
 Rachael Wilkenfeld
PowerScore Staff
  • PowerScore Staff
  • Posts: 1358
  • Joined: Dec 15, 2011
|
#57342
Hi Michael,

On the LSAT we have to generally avoid adding potential additional facts beyond the stimulus, aside those in a help/hurt answer choice. In this case, we have a stimulus where an individual makes a claim. There is no description of supporting evidence, supporting witnesses, or other supporting information. It is true that those things could exist, but they are not a part of this stimulus. This stimulus is what we need to weaken. If we think about what could exist beyond the stimulus, we could easily make faulty assumptions.

So let's look at what answer choice (A) states. If a claim is extraordinary (which, the stimulus states the claim is) it should not be presented uncritically unless there is extraordinary supporting evidence. Here, there's absolutely no evidence given, aside from Mr. Hanlon's word. Therefore, it would weaken the claim of bias to have a principal that required such evidence for an uncritical presentation.

Hope that helps!
 lanereuden
  • Posts: 147
  • Joined: May 30, 2019
|
#67851
Two things,
First with regard to A,
Who are we to say whether or not testimony is “evidence of extraordinarily high standard”
Yes it makes sense it’s not, but what gives us that right to make that assessment

Secondly,
What’s wrong with B?
I imagine intermediate source is not stated nor alluded to in stimulus and so is wrong, at least that is the explanation I read elsewhere.
But to me, isn’t Mr Honlan the intermediate source here? I mean, he saw it, and the newspaper published it, doesn’t that make the newspaper not the true source, i.e. there is intermediate source—that is, Mr Holan?

Get the most out of your LSAT Prep Plus subscription.

Analyze and track your performance with our Testing and Analytics Package.